What are your chances of acceptance?

Calculate for all schools, your chance of acceptance.

Duke University

Your chancing factors

Extracurriculars.

all ap lang argument essay prompts

How to Write the AP Lang Argument Essay + Examples

What’s covered:, what is the ap language argument essay, tips for writing the ap language argument essay, ap english language argument essay examples, how will ap scores impact my college chances.

In 2023, over 550,148 students across the U.S. took the AP English Language and Composition Exam, and 65.2% scored higher than a 3. The AP English Language Exam tests your ability to analyze a piece of writing, synthesize information, write a rhetorical essay, and create a cohesive argument. In this post, we’ll be discussing the best way to approach the argumentative essay section of the test, and we’ll give you tips and tricks so you can write a great essay.

The AP English Language Exam as of 2023 is structured as follows:

Section 1: 45 multiple choice questions to be completed in an hour. This portion counts for 45% of your score. This section requires students to analyze a piece of literature. The questions ask about its content and/or what could be edited within the passage.

Section 2: Three free response questions to be completed in the remaining two hours and 15 minutes. This section counts for 55% of your score. These essay questions include the synthesis essay, the rhetorical essay, and the argumentative essay.

  • Synthesis essay: Read 6-7 sources and create an argument using at least three of the sources.
  • Rhetorical analysis essay: Describe how a piece of writing evokes meaning and symbolism.
  • Argumentative essay: Pick a side of a debate and create an argument based on evidence. In this essay, you should develop a logical argument in support of or against the given statement and provide ample evidence that supports your conclusion. Typically, a five paragraph format is great for this type of writing. This essay is scored holistically from 1 to 9 points.

Do you want more information on the structure of the full exam? Take a look at our in-depth overview of the AP Language and Composition Exam .

Although the AP Language Argument may seem daunting at first, once you understand how the essay should be structured, it will be a lot easier to create cohesive arguments.

Below are some tips to help you as you write the essay.

1. Organize your essay before writing

Instead of jumping right into your essay, plan out what you will say beforehand. It’s easiest to make a list of your arguments and write out what facts or evidence you will use to support each argument. In your outline, you can determine the best order for your arguments, especially if they build on each other or are chronological. Having a well-organized essay is crucial for success.

2. Pick one side of the argument, but acknowledge the other side

When you write the essay, it’s best if you pick one side of the debate and stick with it for the entire essay. All your evidence should be in support of that one side. However, in your introductory paragraph, as you introduce the debate, be sure to mention any merit the arguments of the other side has. This can make the essay a bit more nuanced and show that you did consider both sides before determining which one was better. Often, acknowledging another viewpoint then refuting it can make your essay stronger.

3. Provide evidence to support your claims

The AP readers will be looking for examples and evidence to support your argument. This doesn’t mean that you need to memorize a bunch of random facts before the exam. This just means that you should be able to provide concrete examples in support of your argument.

For example, if the essay topic is about whether the role of the media in society has been detrimental or not, and you argue that it has been, you may talk about the phenomenon of “fake news” during the 2016 presidential election.

AP readers are not looking for perfect examples, but they are looking to see if you can provide enough evidence to back your claim and make it easily understood.

4. Create a strong thesis statement

The thesis statement will set up your entire essay, so it’s important that it is focused and specific, and that it allows for the reader to understand your body paragraphs. Make sure your thesis statement is the very last sentence of your introductory paragraph. In this sentence, list out the key points you will be making in the essay in the same order that you will be writing them. Each new point you mention in your thesis should start a paragraph in your essay.

Below is a prompt and sample student essay from the May 2019 exam . We’ll look at what the student did well in their writing and where they could improve.

Prompt: “The term “overrated” is often used to diminish concepts, places, roles, etc. that the speaker believes do not deserve the prestige they commonly enjoy; for example, many writers have argued that success is overrated, a character in a novel by Anthony Burgess famously describes Rome as a “vastly overrated city,” and Queen Rania of Jordan herself has asserted that “[b]eing queen is overrated.”

Select a concept, place, role, etc. to which you believe that the term “overrated” should be applied. Then, write a well-developed essay in which you explain your judgment. Use appropriate evidence from your reading, experience, or observations to support your argument.

Sample Student Essay #1:

[1] Competition is “overrated.” The notion of motivation between peers has evolved into a source of unnecessary stress and even lack of morals. Whether it be in an academic environment or in the industry, this new idea of competition is harmful to those competing and those around them.

[2] Back in elementary school, competition was rather friendly. It could have been who could do the most pushups or who could get the most imaginary points in a classroom for a prize. If you couldn’t do the most pushups or win that smelly sticker, you would go home and improve yourself – there would be no strong feelings towards anyone, you would just focus on making yourself a better version of yourself. Then as high school rolled around, suddenly applying for college doesn’t seem so far away –GPA seems to be that one stat that defines you – extracurriculars seem to shape you – test scores seem to categorize you. Sleepless nights, studying for the next day’s exam, seem to become more and more frequent. Floating duck syndrome seems to surround you (FDS is where a competitive student pretends to not work hard but is furiously studying beneath the surface just like how a duck furiously kicks to stay afloat). All of your competitors appear to hope you fail – but in the end what do you and your competitor’s gain? Getting one extra point on the test? Does that self-satisfaction compensate for the tremendous amounts of acquired stress? This new type of “competition” is overrated – it serves nothing except a never-ending source of anxiety and seeks to weaken friendships and solidarity as a whole in the school setting.

[3] A similar idea of “competition” can be applied to business. On the most fundamental level, competition serves to be a beneficial regulator of prices and business models for both the business themselves and consumers. However, as businesses grew increasingly greedy and desperate, companies resorted to immoral tactics that only hurt their reputations and consumers as a whole. Whether it be McDonald’s coupons that force you to buy more food or tech companies like Apple intentionally slowing down your iPhone after 3 years to force you to upgrade to the newest device, consumers suffer and in turn speak down upon these companies. Similar to the evolved form of competition in school, this overrated form causes pain for all parties and has since diverged from the encouraging nature that the principle of competition was “founded” on.

The AP score for this essay was a 4/6, meaning that it captured the main purpose of the essay but there were still substantial parts missing. In this essay, the writer did a good job organizing the sections and making sure that their writing was in order according to the thesis statement. The essay first discusses how competition is harmful in elementary school and then discusses this topic in the context of business. This follows the chronological order of somebody’s life and flows nicely.

The arguments in this essay are problematic, as they do not provide enough examples of how exactly competition is overrated. The essay discusses the context in which competition is overrated but does not go far enough in explaining how this connects to the prompt.

In the first example, school stress is used to explain how competition manifests. This is a good starting point, but it does not talk about why competition is overrated; it simply mentions that competition can be unhealthy. The last sentence of that paragraph is the main point of the argument and should be expanded to discuss how the anxiety of school is overrated later on in life. 

In the second example, the writer discusses how competition can lead to harmful business practices, but again, this doesn’t reflect the reason this would be overrated. Is competition really overrated because Apple and McDonald’s force you to buy new products? This example could’ve been taken one step farther. Instead of explaining why business structures—such as monopolies—harm competition, the author should discuss how those particular structures are overrated.

Additionally, the examples the writer used lack detail. A stronger essay would’ve provided more in-depth examples. This essay seemed to mention examples only in passing without using them to defend the argument.

It should also be noted that the structure of the essay is incomplete. The introduction only has a thesis statement and no additional context. Also, there is no conclusion paragraph that sums up the essay. These missing components result in a 4/6.

Now let’s go through the prompt for a sample essay from the May 2022 exam . The prompt is as follows:

Colin Powell, a four-star general and former United States Secretary of State, wrote in his 1995 autobiography: “[W]e do not have the luxury of collecting information indefinitely. At some point, before we can have every possible fact in hand, we have to decide. The key is not to make quick decisions, but to make timely decisions.”

Write an essay that argues your position on the extent to which Powell’s claim about making decisions is valid. 

In your response you should do the following:

  • Respond to the prompt with a thesis that presents a defensible position. 
  • Provide evidence to support your line of reasoning. 
  • Explain how the evidence supports your line of reasoning. 
  • Use appropriate grammar and punctuation in communicating your argument.

Sample Student Essay #2:

Colin Powell, who was a four star general and a former United States Secretary of State. He wrote an autobiography and had made a claim about making decisions. In my personal opinion, Powell’s claim is true to full extent and shows an extremely valuable piece of advice that we do not consider when we make decisions.

Powell stated, “before we can have every possible fact in hand we have to decide…. but to make it a timely decision” (1995). With this statement Powell is telling the audience of his autobiography that it does not necessarily matter how many facts you have, and how many things you know. Being able to have access to everything possible takes a great amount of time and we don’t always have all of the time in the world. A decision has to be made with what you know, waiting for something else to come in while trying to make a decision whether that other fact is good or bad you already have a good amount of things that you know. Everyone’s time is valuable, including yours. At the end of the day the decision will have to be made and that is why it should be made in a “timely” manner.

This response was graded for a score of 2/6. Let’s break down the score to smaller points that signify where the student fell short.

The thesis in this essay is clearly outlined at the end of the first paragraph. The student states their agreement with Powell’s claim and frames the rest of their essay around this stance. The success in scoring here lies in the clear communication of the thesis and the direction the argument will take. It’s important to make the thesis statement concise, specific, and arguable, which the student has successfully done.

While the student did attempt to provide evidence to support their thesis, it’s clear that their explanation lacks specific detail and substance. They referenced Powell’s statement, but did not delve into how this statement has proven true in specific instances, and did not provide examples that could bring the argument to life.

Commentary is an essential part of this section’s score. It means explaining the significance of the evidence and connecting it back to the thesis. Unfortunately, the student’s commentary here is too vague and does not effectively elaborate on how the evidence supports their argument.

To improve, the student could use more concrete examples to demonstrate their point and discuss how each piece of evidence supports their thesis. For instance, they could discuss specific moments in Powell’s career where making a timely decision was more valuable than waiting for all possible facts. This would help illustrate the argument in a more engaging, understandable way.

A high score in the “sophistication” category of the grading rubric is given for demonstrating a complex understanding of the rhetorical situation (purpose, audience, context, etc.), making effective rhetorical choices, or establishing a line of reasoning. Here, the student’s response lacks complexity and sophistication. They’ve simply agreed with Powell’s claim and made a few general statements without providing a deeper analysis or effectively considering the rhetorical situation.

To increase sophistication, the student could explore possible counterarguments or complexities within Powell’s claim. They could discuss potential drawbacks of making decisions without all possible facts, or examine situations where timely decisions might not yield the best results. By acknowledging and refuting these potential counterarguments, they could add more depth to their analysis and showcase their understanding of the complexities involved in decision-making.

The student could also analyze why Powell, given his background and experiences, might have come to such a conclusion, thus providing more context and showing an understanding of the rhetorical situation.

Remember, sophistication in argumentation isn’t about using fancy words or complicated sentences. It’s about showing that you understand the complexity of the issue at hand and that you’re able to make thoughtful, nuanced arguments. Sophistication shows that you can think critically about the topic and make connections that aren’t immediately obvious.

Now that you’ve looked at an example essay and some tips for the argumentative essay, you know how to better prepare for the AP English Language and Composition Exam.

While your AP scores don’t usually impact your admissions chances , colleges do care a lot about your course rigor. So, taking as many APs as you can will certainly boost your chances! AP scores can be a way for high-performing students to set themselves apart, particularly when applying to prestigious universities. Through the process of self-reporting scores , you can show your hard work and intelligence to admissions counselors.

That said, the main benefit of scoring high on AP exams comes once you land at your dream school, as high scores can allow you to “test out” of entry-level requirements, often called GE requirements or distribution requirements. This will save you time and money.

To understand how your course rigor stacks up, check out CollegeVine’s free chancing engine . This resource takes your course rigor, test scores, extracurriculars, and more, to determine your chances of getting into over 1600 colleges across the country!

Related CollegeVine Blog Posts

all ap lang argument essay prompts

all ap lang argument essay prompts

Crafting an Impressive Argumentative Essay for AP Lang

all ap lang argument essay prompts

Writing an argumentative essay for AP Language and Composition requires a strategic approach to effectively convey your perspective. Here's a guide to crafting an impressive argumentative essay:

1. Understand the Prompt:

   - Tip: Carefully read and analyze the prompt. Identify the key elements, including the task you are asked to perform and any specific requirements or constraints.

2. Develop a Clear Thesis Statement:

   - Tip: Formulate a concise and focused thesis statement that clearly states your argument or position. This statement should guide the reader on what to expect in your essay.

3. Identify Your Audience:

   - Tip: Consider your target audience and tailor your language and arguments accordingly. Understanding your audience helps you establish a connection and effectively convey your message.

4. Organize Your Essay Effectively:

   - Tip: Structure your essay with a clear introduction, body paragraphs, and a conclusion. Ensure a logical flow of ideas, with each paragraph contributing to the overall argument.

5. Provide Context and Background:

   - Tip: Begin with a brief introduction that provides context for your argument. Clearly state the issue at hand and offer background information to help readers understand the significance of your argument.

6. Present a Strong Claim:

   - Tip: Clearly articulate your main claim or argument. This should be a debatable statement that forms the core of your essay. Avoid vague or overly broad claims.

7. Support Your Argument with Evidence:

   - Tip: Use relevant and compelling evidence to support your claims. This can include facts, statistics, examples, anecdotes, or quotations. Ensure that your evidence is credible and directly contributes to your argument.

8. Address Counterarguments:

   - Tip: Acknowledge potential counterarguments and address them in your essay. This demonstrates a nuanced understanding of the issue and strengthens your overall argument.

9. Use Persuasive Language:

    - Tip: Choose language that is persuasive and impactful. Utilize rhetorical devices, vivid imagery, and compelling language to engage your readers and convey the urgency or importance of your argument.

10. Craft Well-Reasoned Body Paragraphs:

    - Tip: Dedicate separate paragraphs to different aspects of your argument. Each paragraph should have a clear topic sentence, supporting evidence, and a concise explanation of how the evidence relates to your thesis.

11. Ensure Cohesive Transitions:

    - Tip: Use transitional phrases and sentences to ensure a smooth transition between paragraphs. This enhances the coherence of your essay and guides readers through your line of reasoning.

12. Conclude Effectively:

    - Tip: Summarize your main points in the conclusion and restate the significance of your argument. Avoid introducing new information in the conclusion. End with a strong closing statement that leaves a lasting impression.

13. Revise and Edit:

    - Tip: Set aside time for revision and editing. Review your essay for clarity, coherence, and grammatical accuracy. Ensure that your argument is compelling and well-supported.

14. Seek Feedback:

    - Tip: If possible, seek feedback from peers, teachers, or mentors. External perspectives can provide valuable insights and help you identify areas for improvement.

15. Stay Focused on the Argument:

    - Tip: Throughout the essay, stay focused on your central argument. Avoid unnecessary tangents or diverging from the main point. Every part of your essay should contribute to strengthening your overall argument.

Crafting an impressive argumentative essay for AP Language and Composition involves a combination of persuasive writing techniques, solid evidence, and effective organization. By following these tips, you can create a compelling essay that showcases your ability to engage in sophisticated argumentation and rhetorical analysis.

You Might Also Like

all ap lang argument essay prompts

Planning for Successful College Applications

Know the right way for successful college application and how to get prepared for college admission to gain admission in your dream college - Read our blog

all ap lang argument essay prompts

Apply for Federal Grants for College Education

Federal Grants are the popular sources of funding for higher education. If you are unaware of federal grants for college and how it works, then read our blog.

all ap lang argument essay prompts

Guidelines To Write Impressive High School Resume

Know some important guidelines will help you write an effective high school resume that will stand out in the crowd. Check out resume sample here

AP Guru has been helping students since 2010 gain admissions to their dream universities by helping them in their college admissions and SAT and ACT Prep

Free Resources

logo-type-white

AP® English Language

How to get a 6 on the argument frq in ap® english language.

  • The Albert Team
  • Last Updated On: March 1, 2022

How to Get a 6 on the Argument FRQ in AP® English Language

What We Review

Introduction: How to Get a 6 on the Argument FRQ in AP® English Language

Wondering how to get a 6 on the argumentative essay in AP® English Language? 

To score an 5 on the AP® English Argument FRQ question, the CollegeBoard scoring guidelines outline that students need to write an essay that effectively argues a position, uses appropriate and convincing evidence, and showcases a wide range of the elements of writing. Essays that score a 6 do all of that and, additionally, demonstrate sophistication in their argument.

An essay that does all of that is an incredibly well-constructed essay. Such an essay needs a solid framework and excellent support. To do this, it is important to have a clear idea of what you are being asked, to not waffle, to spend time and care with your thesis and outline, and to support every claim you make.

We know the best way to write an AP® English FRQ that does everything right is to understand what you are going to see on the AP® English Language test. Read on to prepare yourself for exam day and earn that 6!

What to Expect from the AP® English Language Argument Free Response Questions

The AP® English argument FRQ is the most straightforward of the AP® English FRQs because it is the most similar to the essays you’re already used to writing. It’s exciting to have free reign and make your own argument, unrestrained from rhetorical analysis devices or documents. But, like most AP® writing, it also can be a little overwhelming.

There’s nothing to read and analyze to provide evidence or help you form an argument. Whether you’re feeling excited or overwhelmed by the AP® writing argument FRQ, consider the rhetorical situation. Be strategic about forming your thesis, craft a strong, chronological argument, and utilize good, supportive evidence to earn a better overall essay response.

Determine the question.

The first question to ask yourself is what am I being asked to do ? This may seem obvious, but it’s surprising how tricky it can be to figure out. Look for keywords and phrases that will answer that question.

Here’s an example from the 2019 AP® English Language argumentative essay.

What to Expect from the AP® English Language Argument Free Response Questions - Determine the Question

Though there are just two short paragraphs, there is a lot of room for confusion here. In this case, “Then, write a well-developed essay in which you explain your judgment.” is the key sentence you are looking for. In 2019, AP® English Language test takers were asked to select a concept, place, role, etc. that they believed was “overrated,” and explain why.

If you cannot determine what the question is, go back and reread the prompt. Focus on the last few sentences, as that’s where you’ll usually find it.

Knowing the question you are answering is the most important part of AP® writing. You will not be able to answer the question effectively if you aren’t certain what the question is. Pick out a specific sentence or two to determine the question, and thereby ensure that you aren’t just writing an essay that responds to the general sense of the argument essay prompts

Pick an opinion and stick to it.

The next step is both simple and difficult. Identify your own opinion on the subject.

But remember — the AP® argumentative essay exam format is designed to test how well you can craft an argument. Questions like the 2019 question seem so daunting, because claiming anything to be “overrated” is such a broad topic. It is a bigger question than students are used to encountering on an AP® test.

But, always remember, there is no right or wrong answer for this AP® English FRQ. And whatever argument you choose will not come back later in the exam or in your final grade in the class. This is not to say that you shouldn’t believe in what you are writing. Only that you should remember that both sides are arguable, pick one, and stick to it. Don’t waffle.

Below we break down two sample student answers from this same 2019 prompt. 

What to Expect from the AP® English Language Argument Free Response Questions - PIck an opinion negative example

In this AP® Lang argument essay example, the student jumps from describing places, to people, to outfits. The prompts asked for only one example and the student gives three.  By doing this, it shows they were not only unable to grasp what the prompt was asking, but that they couldn’t stick to their opinion.  Instead of deeply strengthening one choice, the student gives vague, half-reasons for too many choices. When writing your FRQs, choose just one concept and stick to it.

The following example demonstrates a strong student response:

What to Expect from the AP® English Language Argument Free Response Questions - Pick an opinion strong example

This student picks one clear concept, capitalism, and clearly outlines their support for it.  They write with clear language that opens the door for the deeper analysis coming later in the essay.

Like this student, choose just one clear argument to delve into when writing your FRQ.

Craft a thesis statement.

The thesis statement should be both simple and elegant. Students often find it one of the more difficult writing skills to master, but we’re here to help. Just remember that it should encompass your entire essay in just one sentence.  So, for the 2019 argument FRQ :

Good thesis: While capitalism undeniably has its upsides, it has many downsides that are rarely recognized. When considering the downsides, capitalism is clearly overrated as it commodifies humanity and uplifts a minority at the expense of the majority.

This thesis breaks down a) that the author clearly states his claim that capitalism is overrated, b) that the author will support that claim with examples on how it commodifies humanity and how it hurts the majority in favor of the minority.

Good thesis: While the Electoral College was created in the name of equality for smaller states, it is ultimately overrated because it undercuts the popular vote, it is an archaic practice that is unsuitable for the modern era.

This thesis claims the Electoral College is overrated by claiming it doesn’t do what it was created to do in the first place- support equality.  It also introduces two supporting examples for the rest of the essay- it undercuts the popular vote and it doesn’t work in the modern era.

Not a good thesis: Kicking a ball in a net and scoring, is not as important as saving lives. Soccer to me would be considered overrated.

This thesis doesn’t give clear direction for the rest of the essay.  The author claims soccer is overrated, but doesn’t tell us why. The example that “it’s not as important as saving lives” is unrelated and also not touched on again later in the essay. This thesis isn’t specific and doesn’t give you a clear idea of what the author will be saying next.

Not a good thesis: The term “overrated” has been used in conversation to diminish the value of roles. In unusual circumstances the term “overrated” should be applied to the idea of freedom in regards to social change, but overall it should not be applied in regards to global devastation and cruel treatment.

This thesis does not directly answer the question.  Is the author arguing that freedom is overrated? They also claim that the term overrated doesn’t apply to global devastation and cruel treatment. This second claim is both unrelated to the first and doesn’t work to answer the initial prompt.

Looking at these four examples, can you see the difference between a strong and weak thesis?

After you’ve determined your thesis, use it as a jumping point to sketch a quick outline. Then, follow your outline, bringing in your own concrete examples and evidence. Doing so will improve your AP® writing.

Return to the Table of Contents

Craft a chronological argument.

A good argument builds as you move through the essay. It does not simply repeat the same points. Instead, the different points of the argument build off one another and work together to advance the author’s point.

Let’s look at the 2018 AP® English argument FRQ for an example.

What to Expect from the AP® English Language Argument Free Response Questions - Craft a chronological argument

In this case, students are being asked to argue a position on the value of choosing the unknown. 

All students are likely to have their own definitions of what “choosing the unknown” might mean. You first want to consider what this phrase means to you, and how it applies to the real world.  Could it mean breaking out of your comfort zone in daily routines, or could it mean going to theater school to follow your dreams?  There’s no wrong answers, but try to pin down one. Consider Lindbergh’s quote the prompt gives you, and how shock, disappointment, and enrichment play into choosing the unknown.

Once you’ve nailed down your definition, you can begin to form your arguments. A chronological argument builds off itself. So, in this question’s case, an outline would look something like this:

  • Choosing the unknown is necessary for the development of the human race.
  • Scientific advancements cannot be made without testing the boundaries of the unknown.
  • Cultural and artistic growth can only occur through exploring the unknown.

First, a student must define what choosing the unknown means, and what makes it difficult. Next the student argues for the value of choosing the unknown, in that the human race could never develop without it.  Finally, the student will argue for the invaluable scientific and cultural/artistic advances made throughout history by breaking known boundaries.

When you sketch your outline, quickly ask yourself if the outline would make just as much sense if you rearranged it. If the answer is no, start writing your essay. If the answer is yes, try to structure your argument so that your points build off one another.

Support your claims.

All arguments need evidence. This is the proof you need to support your thesis. And in the case of the AP® English argument FRQ, the evidence all comes from you. What exactly that evidence is will vary from question to question and from student to student. But make sure that every point you make is supported by evidence.

Here’s some good news — you already know quite a bit about effective evidence from what you have learned in AP® English about rhetorical devices. Your main purpose in this essay is to persuade. What have you learned in class about effective ways to persuade? What rhetorical devices can you utilize? Try to pick the best devices to support your argument that you can.

Here are some examples of supportive and non-supportive evidence that students could use to support their claims.

What to Expect from the AP® English Language Argument Free Response Questions - Support your claims

The 2017 AP® English language argument FRQ asked students to argue a position if the most essential skill is artifice. The example student answers given below are from here .

Supportive evidence:   “Throughout history, rulers have utilized countless different methods of achieving power, however none have been so successful as mastering the art of lying.

In his advice to future rulers, Niccolo Machiavelli encouraged them to lie and maintain the illusion of sympathy to the common struggles in order to retain power. He asserts that it is imperative for a ruler to appear caring and sympathetic even if he has no objective but power.

Machiavelli argues that to be sincere and honest is akin to being vulnerable. A ruler must be skilled in the art of deception if he is not to fall prey to usurpers. Thus, it is essential that he appear humble and morally upright to his constituents as he is to appear idealistic, despite his nature being identical to his citizens.”

In this paragraph, the student chooses to discuss the role of artifice in politics. The student claims that mastering lying is essential to achieving political power. The student uses Machiavelli’s leadership and beliefs as specific examples to support this, by analyzing and connecting each point back to his/her claim.

Non-supportive evidence: “Another example would be actors on red carpets or at interviews they sound generous and relatable, but in reality they could be selfish people who don’t care about anyone. To the public they act charming, honest, and sincere. They do this so they can get famous and rich. They do this so they will never get ignored.”

In this paragraph, the student chooses to discuss the role of artifice in the culture of entertainment and celebrities. However, the student does not utilize supportive evidence to do so. The paragraph is full of claims about how actors lie, but does not provide a concrete example to anchor the claims. The student provides a lot of very vague generalizations, but no clear evidence or examples of specific celebrities and how they used artifice to succeed.

There is so much variance in prompts and students’ prior knowledge; it’s impossible to provide a checklist of what makes evidence supportive. But a good trick to decide if you’ve supported your claims well enough is to talk to yourself. No really, it’s a good idea.

Picture yourself discussing your essay with someone. Imagine that this person disagrees with everything that you say. Every time you make a claim, like that it’s important to be polite in an email, your imaginary person shakes their head and tells you no. How would you try to convince them? What examples would you use? Make sure that for each opinion you put forward; you have provided an answer to someone who would disagree with you.

The evidence is an important part of your essay. If your outline and your argument are a framework, your evidence is the brick and mortar. A house without brick and mortar won’t fall, but it won’t be a very nice house to inhabit. Tie every claim you make to a piece of evidence to ensure the best essay possible.

Wrapping Things Up: Scoring a 6 on the Argument FRQ for AP® English Language

The AP® English argument FRQ varies quite a bit. But it is ultimately about how well you can put forth an argument. So, don’t be afraid to spend some time crafting that argument. We’ve covered a lot in this article- here are the main points to remember:

  • Determine the question. Figure out what the prompt is asking you to do.
  • Pick an opinion and stick to it. Choose one side of the argument and one clear claim to support all the way through.
  • Craft a thesis statement. Your thesis should be clear, concise, and introduce the content of your essay.
  • Craft a chronological argument. Make an argument that builds on its prior points.
  • Support your claims. Support yourself with concrete, specific evidence and examples. 

But most of all, have fun. This essay is the one you should be looking forward to, where you have the freest rein. Enjoy it and earn yourself a 6.

Do the examples shown make sense to you? Can you picture yourself moving through the AP® writing argument FRQ with ease now?

Interested in a school license?​

8 thoughts on “how to get a 6 on the argument frq in ap® english language”.

Thank you for explaining this so eloquently. Excellent post, I will keep this handy and refer to it often from now on. It’s so educative. Great post!

Sure, glad it helped.

I’m an AP® Language teacher and the title of your article caught my eye because the essays aren’t scored on a 0-9 scale anymore. The max score for an essay now is a 6. Essays are now scored in 3 categories: Thesis: 0 or 1 point Evidence and commentary: 0-4 points Sophistication: 0 or 1 point I just wanted to let you know! I saw this was last updated in 2020 and just thought it should reflect the current AP® exam.

Thank you for the heads up! This is an older blog post that must have had something else updated to it this year. We’ve gone ahead and revised the post.

Hi, my AP® Language teacher emphasized on a counterargument at the end of the supporting paragraphs. Could you elaborate on it? Also, how exactly do we get the sophistication point?

Hi Stephanie, thanks for reaching out! Making a solid counter-argument is definitely one way to make sure that you earn the Sophistication point. We recommend having a look at our AP® English Language Review Guide for 2021 for more tips! The College Board’s Free-Response Question and Scoring Information Archive also provides authentic examples of student writing — many of which successfully make counterarguments and rebuttals to earn the Sophistication point.

Hi can I get a 6?

Hi Roy, we certainly believe that earning a 6 on your FRQs is possible with practice and dedication! I’d recommend having a look at our AP® English Language Review Guide for tips and tricks, and you can also browse our AP® English Language and Composition Resource Page and Free Response practice questions for targeted practice.

Comments are closed.

Popular Posts

AP® Physics I score calculator

AP® Score Calculators

Simulate how different MCQ and FRQ scores translate into AP® scores

all ap lang argument essay prompts

AP® Review Guides

The ultimate review guides for AP® subjects to help you plan and structure your prep.

all ap lang argument essay prompts

Core Subject Review Guides

Review the most important topics in Physics and Algebra 1 .

all ap lang argument essay prompts

SAT® Score Calculator

See how scores on each section impacts your overall SAT® score

all ap lang argument essay prompts

ACT® Score Calculator

See how scores on each section impacts your overall ACT® score

all ap lang argument essay prompts

Grammar Review Hub

Comprehensive review of grammar skills

all ap lang argument essay prompts

AP® Posters

Download updated posters summarizing the main topics and structure for each AP® exam.

Interested in a school license?

all ap lang argument essay prompts

Bring Albert to your school and empower all teachers with the world's best question bank for: ➜ SAT® & ACT® ➜ AP® ➜ ELA, Math, Science, & Social Studies aligned to state standards ➜ State assessments Options for teachers, schools, and districts.

Calculate for all schools

Your chance of acceptance, your chancing factors, extracurriculars, what's the best approach for the ap lang argument essay.

Hey everyone! I was wondering how you guys prepare for and structure your argument essay in AP Lang. What types of evidence do you usually use? And any tips on choosing effective examples to support your argument?

Hi there! The AP Lang argument essay is all about making a strong, well-structured argument in a clear and concise manner. Here's a step-by-step approach you can follow and some tips to help you prepare:

1. Understand the prompt: Carefully read the given prompt and make sure you understand what it's asking you to do. Keep an eye out for key terms that might require closer consideration.

2. Plan your essay: After fully understanding the prompt, take a few minutes to plan your essay. Decide on your main argument, and think about how you can best support it with evidence.

3. Choose your evidence: For the AP Lang argument essay, it's important to use a mix of both evidence from the provided sources and your own knowledge. Consider using historical events, real-life examples, and even personal experiences if they're relevant and can effectively support your argument.

4. Create a clear thesis statement: This should be a concise statement that outlines your main argument and how you plan to support it. Make sure your thesis is specific and directly addresses the prompt.

5. Outline your essay: Break down your essay into an introduction, body paragraphs, and a conclusion. Plan which evidence you'll use in each body paragraph, and be sure to vary the types of evidence to make your argument more compelling.

6. Write the introduction: Begin with a hook or an interesting fact that grabs the reader's attention. Then, provide some context for your argument before clearly stating your thesis.

7. Craft clear topic sentences: Each body paragraph should start with a topic sentence that introduces the main point of that paragraph and connects it back to your thesis.

8. Use appropriate transitions: Ensure there's a smooth flow between your paragraphs and ideas by using appropriate transition words and phrases.

9. Address counterarguments: To make your argument more convincing, consider acknowledging valid counterarguments and then refuting them with evidence.

10. Write a strong conclusion: Sum up your main points, restate your thesis, and leave the reader with something thought-provoking or impactful.

To prepare for the AP Lang argument essay, practice your writing skills regularly, read a variety of sources to broaden your knowledge, and analyze sample essays to understand what works and what doesn't. Remember, the key to an effective argument essay is having a clear, focused thesis and supporting it with strong evidence. Happy writing!

About CollegeVine’s Expert FAQ

CollegeVine’s Q&A seeks to offer informed perspectives on commonly asked admissions questions. Every answer is refined and validated by our team of admissions experts to ensure it resonates with trusted knowledge in the field.

Fiveable

Find what you need to study

2024 AP English Language and Composition Exam Guide

12 min read • august 18, 2023

A Q

Your guide to the 2024 AP English Language and Composition exam

We know that studying for your AP exams can be stressful, but Fiveable has your back! We created a study plan to help you crush your AP English Language and Composition exam. This guide will continue to update with information about the 2024 exams, as well as helpful resources to help you do your best on test day.  Unlock Cram Mode  for access to our cram events—students who have successfully passed their AP exams will answer your questions and guide your last-minute studying LIVE! And don't miss out on unlimited access to our database of thousands of practice questions. FYI, something cool is coming your way Fall 2023! 👀

Format of the 2024 AP English Language and Composition exam

This year, all AP exams will cover all units and essay types. The 2024 AP English Language and Composition exam format will be:

Section I: Multiple Choice - 45% of your score

45 questions in 1 hour

Section II: Free Response Section - 55% of your score

2 hours and 15 minutes for:

1 synthesis essay

1 rhetorical analysis essay

1 argument essay

Scoring Rubric for the 2024 AP Lang Essays

Synthesis Essay

1 point for a defensible thesis that responds to the prompt

Evidence and Commentary

Max of 4 points for providing evidence from at least 3 sources that support the line of reasoning AND commentary that explains and analyzes the evidence

Sophistication

1 point any of the following:

Creating a nuanced argument

Showing the limitations of the argument

Making effective rhetorical choices

Employing a style that is vivid and persuasive

Rhetorical Analysis Essay

1 point for a defensible thesis that analyzes rhetorical choices

Max of 4 points for providing specific evidence AND consistently explaining how the evidence relates to the line of reasoning AND showing how the rhetorical choices contribute to the author's message .

1 point for any of the following:

Explaining the significance of the rhetorical choices ( rhetorical situation )

Explaining the complexities of the passage and their purpose

Argument Essay

1 point for a defensible thesis

Max of 4 points for providing specific evidence AND consistently explaining the relevance of that evidence .

Crafting a nuanced argument by identifying complexities

Explaining the limitations of the argument by placing it in a broader context

Making rhetorical choices to improve the argument

Check out our study plan below to find resources and tools to prepare for your AP English Language and Composition exam.

When is the 2024 AP English Language and Composition Exam and How Do I Take It?

How should i prepare for the ap lang exam.

First, take stock of your progress in the course so far. What areas have you excelled and which sections need more focus? Download the AP English Language Cheatsheet PDF - a single sheet that covers everything you need to know at a high level. Take note of your strengths and weaknesses!

Build your study plan to review every unit and question type, but focus most on the areas that need the most improvement and practice. We’ve put together this plan to help you study between now and May. This will cover all of the units and essay types to prepare you for your exam

Practice essays are your best friends! The more essays you write, the more automatic the process will come, and the easier the AP exam will be!

Try some of the past exam questions here

We've put together the study plan found below to help you study between now and May. This will cover all of the units and essay types to prepare you for your exam. Pay special attention to the units that you need the most improvement in.

Study, practice, and review for test day with other students during our live cram sessions via  Cram Mode . Cram live streams will teach, review, and practice important topics from AP courses, college admission tests, and college admission topics. These streams are hosted by experienced students who know what you need to succeed.

Pre-Work: Set Up Your Study Environment

Before you begin studying, take some time to get organized.

🖥 Create a study space.

Make sure you have a designated place at home to study. Somewhere you can keep all of your materials, where you can focus on learning, and where you are comfortable. Spend some time prepping the space with everything you need and you can even let others in the family know that this is your study space. 

📚 Organize your study materials.

Get your notebook, textbook, prep books, or whatever other physical materials you have. Also, create a space for you to keep track of review. Start a new section in your notebook to take notes or start a Google Doc to keep track of your notes. Get yourself set up!

📅 Plan designated times for studying.

The hardest part about studying from home is sticking to a routine. Decide on one hour every day that you can dedicate to studying. This can be any time of the day, whatever works best for you. Set a timer on your phone for that time and really try to stick to it. The routine will help you stay on track.

🏆 Decide on an accountability plan.

How will you hold yourself accountable to this study plan? You may or may not have a teacher or rules set up to help you stay on track, so you need to set some for yourself. First, set your goal. This could be studying for x number of hours or getting through a unit. Then, create a reward for yourself. If you reach your goal, then x. This will help stay focused!

🤝 Get support from your peers.  

There are thousands of students all over the world who are preparing for their AP exams just like you! Join  Rooms  🤝 to chat, ask questions, and meet other students who are also studying for the spring exams. You can even build study groups and review material together! 

2024 AP Lang Study Guide

🚧 unit 1 foundations of rhetoric: analysis of the rhetorical situation and claims ., big takeaways:.

Unit 1 is an introductory unit that lays the foundations for the reading skills associated with how to understand and analyze complex texts. Skills here include identifying the ASPECTS of a text, analyzing the claim given and the evidence used to support that claim, and determining the function of the “chunks” in the argument. Because the content in this unit is very foundational, it is looped throughout the rest of the course instruction.

Definitely do this:

📚 Read these study guides:

Unit 1 Overview: Claims , Reasoning , and Evidence

1.1 Identifying the purpose and intended audience of a text

1.2 Examining how evidence supports a claim

1.3 Developing paragraphs as part of an effective argument

🎥 Watch these videos:

College Board’s Instructional Video: Overview of The Rhetorical Situation .

Fiveable’s How to Read Like an AP Student .

Rhetorical Analysis Thesis Statements  

Rhetorical Analysis Body Paragraphs

✍️ Practice:

Use the Fiveable ASPECTS Guidesheet to help you break down a complex text.

🗺 Can you identify these rhetorical devices?

You won’t be asked to name drop on the exam, but it can be helpful to use devices when discussing strategies. Try this Quizlet to help prepare.

Unit 2 Foundations of Argument: Analysis of an author’s choices in appeals and evidence

Unit 2 is an introductory unit that builds onto the foundations of rhetorical ASPECTS and moves toward planning and writing your own arguments. This unit focuses on the relationships between subject, speaker, and message, including examination of the structure and purpose of the given argument. The unit then moves into the developing thesis statements and building your own arguments with a clear line of reasoning .

Unit 2 Overview: Organizing Information for a Specific Audience

2.1 Analyzing audience and its relationship to the purpose of an argument

2.2 Building an argument with relevant and strategic evidence

2.3 Developing thesis statements

2.4 Developing structure and integrating evidence to reflect a line of reasoning

College Board’s Instructional Video: Identify Rhetorical Situation in a Pre 20th Century Text .

Fiveable’s video on How to Find Rhetorical Devices  

📰 Check out these articles:

Here’s a list of recommended rhetorical devices with definitions and examples!

Use the Fiveable Rhetorical Precis Guidesheet to help you break down a complex text.

🗺 Can you identify these elements of practical argument?

You won’t be asked to name drop of the exam, but it can be helpful to use devices when discussing strategies. Try this Quizlet to help prepare.

👥 Unit 3 Confluence: Synthesis of multiple sources in argumentation

Unit 3 approaches multiple perspectives in argument through the lens of synthesis (that’s FRQ 1). In this study, you learn to identify effective and faulty reasoning while integrating a variety of evidence from credible resources that is properly cited in an original text.

Unit 3 Overview: Perspectives and How Arguments Relate

3.1 Interpreting character description and perspective

3.2 Identifying and avoiding flawed lines of reasoning

3.3 Introducing and integrating sources and evidence

3.4 Using sufficient evidence for an argument

3.5 Attributing and citing references

3.6 Developing parts of a text with cause-effect and narrative methods

Fiveable’s Introduction into Synthesis Essays and How to Begin Your Argument

College Board’s Instructional Video: Complexity in Argument .

🗺 Can you identify these elements of synthesis?

👀 Unit 4 Reasoning : Analysis of argument from introduction to conclusion

Unit 4 includes a greater depth of focus on the writing of effective arguments -- the line of reasoning created in the introduction, built with modes of discourse, and strengthened in the conclusion. An important note about these skills of argumentation is that they build toward all parts of every FRQ. 

Unit 4 Overview: How writers develop arguments, intros, and conclusion

4.1 Developing and connecting thesis statements and lines of reasoning

4.2 Developing introductions and conclusions

4.3 Adjusting an argument to address new evidence

College Board’s Instructional Video: Understanding a Line of Reasoning .

Fiveable’s Effective Annotations .

Try Fiveable’s Guide to LOR Body Paragraphs .

🗺 Can you identify the rhetorical modes?

You won’t be asked to name drop them on the exam, but it can be helpful to use devices when discussing strategies. Try this Quizlet to help prepare.

🧐 Unit 5 Commentary and Analysis: Analysis of complex argument and intentional rhetoric

In Unit 5, the skills look at the minutiae involved in argumentation: development of the line of reasoning that produces strong commentary and maintains the primary claim through all parts of the writing. To achieve these goals, this unit includes a focus on transitions , modifiers , and qualifications for argumentative perspective .  

Unit 5 Overview

5.1 Maintaining ideas throughout an argument

5.2 Developing commentary throughout paragraphs

5.3 Using modifiers to qualify an argument and convey perspective

5.4 Using transitions

Fiveable’s video on How to Improve Analysis Part 1 and Part 2

As well as how to Embed Quotes into Body Paragraphs  

Rhetorical Analysis Body Paragraphs  

Synthesis Essay Body Paragraphs  

Argument Essay Body Paragraphs

Tara Seale’s adaptation for Creating a Line of Reasoning .

🏃‍♂️ Unit 6 Rhetorical Risks: Analysis of multiple perspectives , bias , and shifts with new evidence

In Unit 6, you will notice a direct link building on the ideas of Unit 3 as this instruction looks at position and perspectives while synthesizing information strategically to support a claim.  For greater depth, this unit moves to modify a current argument to include new evidence .

Unit 6 Overview: Position, Perspective , and Bias

6.1 Incorporating multiple perspectives strategically into an argument

6.2 Recognizing and accounting for bias

6.3 Adjusting an argument to new evidence

6.4 Analyzing tone and shifts in tone

College Board’s Instructional Video: Creating a Nuanced Argument .

Fiveable’s video on Tracking an Author’s Argument  

🚀 Unit 7 Complex Argumentation: Analysis of effective arguments, including concession and refutation

The skills of Unit 7 are about putting all units of study together to look at the complexity of a given argument and the effectiveness of the pieces built into that argument.  Though many teachers will have addressed counterarguments, concessions, and refutations before reaching this unit, those skills are highly scrutinized in this segment of learning.

Unit 7 Overview: Successful and Unsuccessful Arguments

7.1 Examining complexities in issues

7.2 Considering how words, phrases, and clauses can modify and limit an argument

7.3 Examining how counterargument or alternative perspectives affect an argument

7.4 Exploring how sentence development affects an argument

Fiveable’s video on Arguments and Counterarguments  

College Board’s Instructional Video: How Argument Demonstrates Understanding .

Check your progress with Fiveable’s AP Language Skills Matrix .

📝 Unit 8 Style: Analysis of how style influences the audience movement

Unit 8 covers how to understand the influence style has on the audience , and the purpose behind each decision. By analyzing these various tactics, students are able to understand the author’s audience , and how to effectively persuade them. Style is an important part in connecting the rest of the course and understanding how the rhetorical choices and devices are used to accomplish a purpose .

Unit 8 Overview: Stylistic Choices

8.1 Choosing comparisons based on an audience

8.2 Considering how sentence development and word choice affect how the writer is perceived by an audience

8.3 Considering how all choices made in an argument affect the audience

8.4 Considering how style affects an argument

Fiveable’s Analysis of the Mindset of the Audience

College Board’s Instructional video: Analyzing and Understanding the Audience

College Board’s explanation of Elements and Context for Style  

Review this quizlet on Elements of Style for more practice.

✏️ Unit 9 Craft: Creation of your own complex argument with synthesis and rhetoric

The final unit of AP Language and Composition covers how to effectively form your own arguments by acknowledging and understanding complexities to create a nuanced and sophisticated argument. It focuses on your ability to comprehend and connect multiple sources to create a well reasoned, and detailed argument as well as how to add in your own rhetorical devices and choices to make your writing more persuasive and effective.

Unit 9 Overview: Developing a Complex Argument

9.1 Strategically conceding, rebutting, or refuting information

9.2 Crafting an argument through stylistic choices like word choice and description

Fiveable’s video on Creating your own Synthesis Arguments

College Board’s video on Complexities within Arguments and How to Create a Nuanced Argument

Key Terms to Review ( 38 )

Argument Structure

Author's Message

Cause-Effect Method

Comparisons

Conclusions

Counterargument

Introductions

Line of Reasoning

Multiple Perspectives

Narrative Method

Objective Reasoning

Perspective

Qualifications

Rhetorical Choices

Rhetorical Situation

Sentence Development

Stylistic Choices

Subjective Reasoning

Textual Evidence

Thesis Development

Thesis Statement

Tone Shifts

Transitions

Word Choice

Fiveable

Stay Connected

© 2024 Fiveable Inc. All rights reserved.

AP® and SAT® are trademarks registered by the College Board, which is not affiliated with, and does not endorse this website.

PrepScholar

Choose Your Test

Sat / act prep online guides and tips, how to write a perfect synthesis essay for the ap language exam.

author image

Advanced Placement (AP)

body-pencil-sharpen-notebook-1

If you're planning to take the AP Language (or AP Lang) exam , you might already know that 55% of your overall exam score will be based on three essays. The first of the three essays you'll have to write on the AP Language exam is called the "synthesis essay." If you want to earn full points on this portion of the AP Lang Exam, you need to know what a synthesis essay is and what skills are assessed by the AP Lang synthesis essay.

In this article, we'll explain the different aspects of the AP Lang synthesis essay, including what skills you need to demonstrate in your synthesis essay response in order to achieve a good score. We'll also give you a full breakdown of a real AP Lang Synthesis Essay prompt, provide an analysis of an AP Lang synthesis essay example, and give you four tips for how to write a synthesis essay.

Let's get started by taking a closer look at how the AP Lang synthesis essay works!

Synthesis Essay AP Lang: What It Is and How It Works

The AP Lang synthesis essay is the first of three essays included in the Free Response section of the AP Lang exam.

The AP Lang synthesis essay portion of the Free Response section lasts for one hour total . This hour consists of a recommended 15 minute reading period and a 40 minute writing period. Keep in mind that these time allotments are merely recommendations, and that exam takers can parse out the allotted 60 minutes to complete the synthesis essay however they choose.

Now, here's what the structure of the AP Lang synthesis essay looks like. The exam presents six to seven sources that are organized around a specific topic (like alternative energy or eminent domain, which are both past synthesis exam topics).

Of these six to seven sources, at least two are visual , including at least one quantitative source (like a graph or pie chart, for example). The remaining four to five sources are print text-based, and each one contains approximately 500 words.

In addition to six to seven sources, the AP Lang exam provides a written prompt that consists of three paragraphs. The prompt will briefly explain the essay topic, then present a claim that students will respond to in an essay that synthesizes material from at least three of the sources provided.

Here's an example prompt provided by the College Board:

Directions : The following prompt is based on the accompanying six sources.

This question requires you to integrate a variety of sources into a coherent, well-written essay. Refer to the sources to support your position; avoid mere paraphrase or summary. Your argument should be central; the sources should support this argument .

Remember to attribute both direct and indirect citations.

Introduction

Television has been influential in United States presidential elections since the 1960's. But just what is this influence, and how has it affected who is elected? Has it made elections fairer and more accessible, or has it moved candidates from pursuing issues to pursuing image?

Read the following sources (including any introductory information) carefully. Then, in an essay that synthesizes at least three of the sources for support, take a position that defends, challenges, or qualifies the claim that television has had a positive impact on presidential elections.

Refer to the sources as Source A, Source B, etc.; titles are included for your convenience.

Source A (Campbell) Source B (Hart and Triece) Source C (Menand) Source D (Chart) Source E (Ranney) Source F (Koppel)

Like we mentioned earlier, this prompt gives you a topic — which it briefly explains — then asks you to take a position. In this case, you'll have to choose a stance on whether television has positively or negatively affected U.S. elections. You're also given six sources to evaluate and use in your response. Now that you have everything you need, now your job is to write an amazing synthesis essay.

But what does "synthesize" mean, exactly? According to the CollegeBoard, when an essay prompt asks you to synthesize, it means that you should "combine different perspectives from sources to form a support of a coherent position" in writing. In other words, a synthesis essay asks you to state your claim on a topic, then highlight the relationships between several sources that support your claim on that topic. Additionally, you'll need to cite specific evidence from your sources to prove your point.

The synthesis essay counts for six of the total points on the AP Lang exam . Students can receive 0-1 points for writing a thesis statement in the essay, 0-4 based on incorporation of evidence and commentary, and 0-1 points based on sophistication of thought and demonstrated complex understanding of the topic.

You'll be evaluated based on how effectively you do the following in your AP Lang synthesis essay:

Write a thesis that responds to the exam prompt with a defensible position

Provide specific evidence that to support all claims in your line of reasoning from at least three of the sources provided, and clearly and consistently explain how the evidence you include supports your line of reasoning

Demonstrate sophistication of thought by either crafting a thoughtful argument, situating the argument in a broader context, explaining the limitations of an argument

Make rhetorical choices that strengthen your argument and/or employ a vivid and persuasive style throughout your essay.

If your synthesis essay meets the criteria above, then there's a good chance you'll score well on this portion of the AP Lang exam!

If you're looking for even more information on scoring, the College Board has posted the AP Lang Free Response grading rubric on its website. ( You can find it here. ) We recommend taking a close look at it since it includes additional details about the synthesis essay scoring.

body-chisel-break-apart

Don't be intimidated...we're going to teach you how to break down even the hardest AP synthesis essay prompt.

Full Breakdown of a Real AP Lang Synthesis Essay Prompt

In this section, we'll teach you how to analyze and respond to a synthesis essay prompt in five easy steps, including suggested time frames for each step of the process.

Step 1: Analyze the Prompt

The very first thing to do when the clock starts running is read and analyze the prompt. To demonstrate how to do this, we'll look at the sample AP Lang synthesis essay prompt below. This prompt comes straight from the 2018 AP Lang exam:

Eminent domain is the power governments have to acquire property from private owners for public use. The rationale behind eminent domain is that governments have greater legal authority over lands within their dominion than do private owners. Eminent domain has been instituted in one way or another throughout the world for hundreds of years.

Carefully read the following six sources, including the introductory information for each source. Then synthesize material from at least three of the sources and incorporate it into a coherent, well-developed essay that defends, challenges, or qualifies the notion that eminent domain is productive and beneficial.

Your argument should be the focus of your essay. Use the sources to develop your argument and explain the reasoning for it. Avoid merely summarizing the sources. Indicate clearly which sources you are drawing from, whether through direct quotation, paraphrase, or summary. You may cite the sources as Source A, Source B, etc., or by using the descriptions in parentheses.

On first read, you might be nervous about how to answer this prompt...especially if you don't know what eminent domain is! But if you break the prompt down into chunks, you'll be able to figure out what the prompt is asking you to do in no time flat.

To get a full understanding of what this prompt wants you to do, you need to identify the most important details in this prompt, paragraph by paragraph. Here's what each paragraph is asking you to do:

  • Paragraph 1: The prompt presents and briefly explains the topic that you'll be writing your synthesis essay about. That topic is the concept of eminent domain.
  • Paragraph 2: The prompt presents a specific claim about the concept of eminent domain in this paragraph: Eminent domain is productive and beneficial. This paragraph instructs you to decide whether you want to defend, challenge, or qualify that claim in your synthesis essay , and use material from at least three of the sources provided in order to do so.
  • Paragraph 3: In the last paragraph of the prompt, the exam gives you clear instructions about how to approach writing your synthesis essay . First, make your argument the focus of the essay. Second, use material from at least three of the sources to develop and explain your argument. Third, provide commentary on the material you include, and provide proper citations when you incorporate quotations, paraphrases, or summaries from the sources provided.

So basically, you'll have to agree with, disagree with, or qualify the claim stated in the prompt, then use at least three sources substantiate your answer. Since you probably don't know much about eminent domain, you'll probably decide on your position after you read the provided sources.

To make good use of your time on the exam, you should spend around 2 minutes reading the prompt and making note of what it's asking you to do. That will leave you plenty of time to read the sources provided, which is the next step to writing a synthesis essay.

Step 2: Read the Sources Carefully

After you closely read the prompt and make note of the most important details, you need to read all of the sources provided. It's tempting to skip one or two sources to save time--but we recommend you don't do this. That's because you'll need a thorough understanding of the topic before you can accurately address the prompt!

For the sample exam prompt included above, there are six sources provided. We're not going to include all of the sources in this article, but you can view the six sources from this question on the 2018 AP Lang exam here . The sources include five print-text sources and one visual source, which is a cartoon.

As you read the sources, it's important to read quickly and carefully. Don't rush! Keep your pencil in hand to quickly mark important passages that you might want to use as evidence in your synthesis. While you're reading the sources and marking passages, you want to think about how the information you're reading influences your stance on the issue (in this case, eminent domain).

When you finish reading, take a few seconds to summarize, in a phrase or sentence, whether the source defends, challenges, or qualifies whether eminent domain is beneficial (which is the claim in the prompt) . Though it might not feel like you have time for this, it's important to give yourself these notes about each source so you know how you can use each one as evidence in your essay.

Here's what we mean: say you want to challenge the idea that eminent domain is useful. If you've jotted down notes about each source and what it's saying, it will be easier for you to pull the relevant information into your outline and your essay.

So how much time should you spend reading the provided sources? The AP Lang exam recommends taking 15 minutes to read the sources . If you spend around two of those minutes reading and breaking down the essay prompt, it makes sense to spend the remaining 13 minutes reading and annotating the sources.

If you finish reading and annotating early, you can always move on to drafting your synthesis essay. But make sure you're taking your time and reading carefully! It's better to use a little extra time reading and understanding the sources now so that you don't have to go back and re-read the sources later.

body-weightlifting-lift-strong

A strong thesis will do a lot of heavy lifting in your essay. (See what we did there?)

Step 3: Write a Strong Thesis Statement

After you've analyzed the prompt and thoroughly read the sources, the next thing you need to do in order to write a good synthesis essay is write a strong thesis statement .

The great news about writing a thesis statement for this synthesis essay is that you have all the tools you need to do it at your fingertips. All you have to do in order to write your thesis statement is decide what your stance is in relationship to the topic provided.

In the example prompt provided earlier, you're essentially given three choices for how to frame your thesis statement: you can either defend, challenge, or qualify a claim that's been provided by the prompt, that eminent domain is productive and beneficial . Here's what that means for each option:

If you choose to defend the claim, your job will be to prove that the claim is correct . In this case, you'll have to show that eminent domain is a good thing.

If you choose to challenge the claim, you'll argue that the claim is incorrect. In other words, you'll argue that eminent domain isn't productive or beneficial.

If you choose to qualify, that means you'll agree with part of the claim, but disagree with another part of the claim. For instance, you may argue that eminent domain can be a productive tool for governments, but it's not beneficial for property owners. Or maybe you argue that eminent domain is useful in certain circumstances, but not in others.

When you decide whether you want your synthesis essay to defend, challenge, or qualify that claim, you need to convey that stance clearly in your thesis statement. You want to avoid simply restating the claim provided in the prompt, summarizing the issue without making a coherent claim, or writing a thesis that doesn't respond to the prompt.

Here's an example of a thesis statement that received full points on the eminent domain synthesis essay:

Although eminent domain can be misused to benefit private interests at the expense of citizens, it is a vital tool of any government that intends to have any influence on the land it governs beyond that of written law.

This thesis statement received full points because it states a defensible position and establishes a line of reasoning on the issue of eminent domain. It states the author's position (that some parts of eminent domain are good, but others are bad), then goes on to explain why the author thinks that (it's good because it allows the government to do its job, but it's bad because the government can misuse its power.)

Because this example thesis statement states a defensible position and establishes a line of reasoning, it can be elaborated upon in the body of the essay through sub-claims, supporting evidence, and commentary. And a solid argument is key to getting a six on your synthesis essay for AP Lang!

Looking for help studying for your AP exam?

Our one-on-one online AP tutoring services can help you prepare for your AP exams. Get matched with a top tutor who got a high score on the exam you're studying for!

Get a 5 On Your AP Exam

Step 4: Create a Bare-Bones Essay Outline

Once you've got your thesis statement drafted, you have the foundation you need to develop a bare bones outline for your synthesis essay. Developing an outline might seem like it's a waste of your precious time, but if you develop your outline well, it will actually save you time when you start writing your essay.

With that in mind, we recommend spending 5 to 10 minutes outlining your synthesis essay . If you use a bare-bones outline like the one below, labeling each piece of content that you need to include in your essay draft, you should be able to develop out the most important pieces of the synthesis before you even draft the actual essay.

To help you see how this can work on test day, we've created a sample outline for you. You can even memorize this outline to help you out on test day! In the outline below, you'll find places to fill in a thesis statement, body paragraph topic sentences, evidence from the sources provided, and commentary :

  • Present the context surrounding the essay topic in a couple of sentences (this is a good place to use what you learned about the major opinions or controversies about the topic from reading your sources).
  • Write a straightforward, clear, and concise thesis statement that presents your stance on the topic
  • Topic sentence presenting first supporting point or claim
  • Evidence #1
  • Commentary on Evidence #1
  • Evidence #2 (if needed)
  • Commentary on Evidence #2 (if needed)
  • Topic sentence presenting second supporting point or claim
  • Topic sentence presenting three supporting point or claim
  • Sums up the main line of reasoning that you developed and defended throughout the essay
  • Reiterates the thesis statement

Taking the time to develop these crucial pieces of the synthesis in a bare-bones outline will give you a map for your final essay. Once you have a map, writing the essay will be much easier.

Step 5: Draft Your Essay Response

The great thing about taking a few minutes to develop an outline is that you can develop it out into your essay draft. After you take about 5 to 10 minutes to outline your synthesis essay, you can use the remaining 30 to 35 minutes to draft your essay and review it.

Since you'll outline your essay before you start drafting, writing the essay should be pretty straightforward. You'll already know how many paragraphs you're going to write, what the topic of each paragraph will be, and what quotations, paraphrases, or summaries you're going to include in each paragraph from the sources provided. You'll just have to fill in one of the most important parts of your synthesis—your commentary.

Commentaries are your explanation of why your evidence supports the argument you've outlined in your thesis. Your commentary is where you actually make your argument, which is why it's such a critical part of your synthesis essay.

When thinking about what to say in your commentary, remember one thing the AP Lang synthesis essay prompt specifies: don't just summarize the sources. Instead, as you provide commentary on the evidence you incorporate, you need to explain how that evidence supports or undermines your thesis statement . You should include commentary that offers a thoughtful or novel perspective on the evidence from your sources to develop your argument.

One very important thing to remember as you draft out your essay is to cite your sources. The AP Lang exam synthesis essay prompt indicates that you can use generic labels for the sources provided (e.g. "Source 1," "Source 2," "Source 3," etc.). The exam prompt will indicate which label corresponds with which source, so you'll need to make sure you pay attention and cite sources accurately. You can cite your sources in the sentence where you introduce a quote, summary, or paraphrase, or you can use a parenthetical citation. Citing your sources affects your score on the synthesis essay, so remembering to do this is important.

body-green-arrow-down

Keep reading for a real-life example of a great AP synthesis essay response!

Real-Life AP Synthesis Essay Example and Analysis

If you're still wondering how to write a synthesis essay, examples of real essays from past AP Lang exams can make things clearer. These real-life student AP synthesis essay responses can be great for helping you understand how to write a synthesis essay that will knock the graders' socks off .

While there are multiple essay examples online, we've chosen one to take a closer look at. We're going to give you a brief analysis of one of these example student synthesis essays from the 2019 AP Lang Exam below!

Example Synthesis Essay AP Lang Response

To get started, let's look at the official prompt for the 2019 synthesis essay:

In response to our society's increasing demand for energy, large-scale wind power has drawn attention from governments and consumers as a potential alternative to traditional materials that fuel our power grids, such as coal, oil, natural gas, water, or even newer sources such as nuclear or solar power. Yet the establishment of large-scale, commercial-grade wind farms is often the subject of controversy for a variety of reasons.

Carefully read the six sources, found on the AP English Language and Composition 2019 Exam (Question 1), including the introductory information for each source. Write an essay that synthesizes material from at least three of the sources and develops your position on the most important factors that an individual or agency should consider when deciding whether to establish a wind farm.

Source A (photo) Source B (Layton) Source C (Seltenrich) Source D (Brown) Source E (Rule) Source F (Molla)

In your response you should do the following:

  • Respond to the prompt with a thesis presents a defensible position.
  • Select and use evidence from at least 3 of the provided sources to support your line of reasoning. Indicate clearly the sources used through direct quotation, paraphrase, or summary. Sources may be cited as Source A, Source B, etc., or by using the description in parentheses.
  • Explain how the evidence supports your line of reasoning.
  • Use appropriate grammar and punctuation in communicating your argument.

Now that you know exactly what the prompt asked students to do on the 2019 AP Lang synthesis essay, here's an AP Lang synthesis essay example, written by a real student on the AP Lang exam in 2019:

[1] The situation has been known for years, and still very little is being done: alternative power is the only way to reliably power the changing world. The draw of power coming from industry and private life is overwhelming current sources of non-renewable power, and with dwindling supplies of fossil fuels, it is merely a matter of time before coal and gas fuel plants are no longer in operation. So one viable alternative is wind power. But as with all things, there are pros and cons. The main factors for power companies to consider when building wind farms are environmental boon, aesthetic, and economic factors.

[2] The environmental benefits of using wind power are well-known and proven. Wind power is, as qualified by Source B, undeniably clean and renewable. From their production requiring very little in the way of dangerous materials to their lack of fuel, besides that which occurs naturally, wind power is by far one of the least environmentally impactful sources of power available. In addition, wind power by way of gearbox and advanced blade materials, has the highest percentage of energy retention. According to Source F, wind power retains 1,164% of the energy put into the system – meaning that it increases the energy converted from fuel (wind) to electricity 10 times! No other method of electricity production is even half that efficient. The efficiency and clean nature of wind power are important to consider, especially because they contribute back to power companies economically.

[3] Economically, wind power is both a boon and a bone to electric companies and other users. For consumers, wind power is very cheap, leading to lower bills than from any other source. Consumers also get an indirect reimbursement by way of taxes (Source D). In one Texan town, McCamey, tax revenue increased 30% from a wind farm being erected in the town. This helps to finance improvements to the town. But, there is no doubt that wind power is also hurting the power companies. Although, as renewable power goes, wind is incredibly cheap, it is still significantly more expensive than fossil fuels. So, while it is helping to cut down on emissions, it costs electric companies more than traditional fossil fuel plants. While the general economic trend is positive, there are some setbacks which must be overcome before wind power can take over as truly more effective than fossil fuels.

[4] Aesthetics may be the greatest setback for power companies. Although there may be significant economic and environmental benefit to wind power, people will always fight to preserve pure, unspoiled land. Unfortunately, not much can be done to improve the visual aesthetics of the turbines. White paint is the most common choice because it "[is] associated with cleanliness." (Source E). But, this can make it stand out like a sore thumb, and make the gargantuan machines seem more out of place. The site can also not be altered because it affects generating capacity. Sound is almost worse of a concern because it interrupts personal productivity by interrupting people's sleep patterns. One thing for power companies to consider is working with turbine manufacturing to make the machines less aesthetically impactful, so as to garner greater public support.

[5] As with most things, wind power has no easy answer. It is the responsibility of the companies building them to weigh the benefits and the consequences. But, by balancing economics, efficiency, and aesthetics, power companies can create a solution which balances human impact with environmental preservation.

And that's an entire AP Lang synthesis essay example, written in response to a real AP Lang exam prompt! It's important to remember AP Lang exam synthesis essay prompts are always similarly structured and worded, and students often respond in around the same number of paragraphs as what you see in the example essay response above.

Next, let's analyze this example essay and talk about what it does effectively, where it could be improved upon, and what score past exam scorers awarded it.

To get started on an analysis of the sample synthesis essay, let's look at the scoring commentary provided by the College Board:

  • For development of thesis, the essay received 1 out of 1 possible points
  • For evidence and commentary, the essay received 4 out of 4 possible points
  • For sophistication of thought, the essay received 0 out of 1 possible points.

This means that the final score for this example essay was a 5 out of 6 possible points . Let's look more closely at the content of the example essay to figure out why it received this score breakdown.

Thesis Development

The thesis statement is one of the three main categories that is taken into consideration when you're awarded points on this portion of the exam. This sample essay received 1 out of 1 total points.

Now, here's why: the thesis statement clearly and concisely conveys a position on the topic presented in the prompt--alternative energy and wind power--and defines the most important factors that power companies should consider when deciding whether to establish a wind farm.

Evidence and Commentary

The second key category taken into consideration when synthesis exams are evaluated is incorporation of evidence and commentary. This sample received 4 out of 4 possible points for this portion of the synthesis essay. At bare minimum, this sample essay meets the requirement mentioned in the prompt that the writer incorporate evidence from at least three of the sources provided.

On top of that, the writer does a good job of connecting the incorporated evidence back to the claim made in the thesis statement through effective commentary. The commentary in this sample essay is effective because it goes beyond just summarizing what the provided sources say. Instead, it explains and analyzes the evidence presented in the selected sources and connects them back to supporting points the writer makes in each body paragraph.

Finally, the writer of the essay also received points for evidence and commentary because the writer developed and supported a consistent line of reasoning throughout the essay . This line of reasoning is summed up in the fourth paragraph in the following sentence: "One thing for power companies to consider is working with turbine manufacturing to make the machines less aesthetically impactful, so as to garner greater public support."

Because the writer did a good job consistently developing their argument and incorporating evidence, they received full marks in this category. So far, so good!

Sophistication of Thought

Now, we know that this essay received a score of 5 out of 6 total points, and the place where the writer lost a point was on the basis of sophistication of thought, for which the writer received 0 out of 1 points. That's because this sample essay makes several generalizations and vague claims where it could have instead made specific claims that support a more balanced argument.

For example, in the following sentence from the 5th paragraph of the sample essay, the writer misses the opportunity to state specific possibilities that power companies should consider for wind energy . Instead, the writer is ambiguous and non-committal, saying, "As with most things, wind power has no easy answer. It is the responsibility of the companies building them to weigh the benefits and consequences."

If the writer of this essay was interested in trying to get that 6th point on the synthesis essay response, they could consider making more specific claims. For instance, they could state the specific benefits and consequences power companies should consider when deciding whether to establish a wind farm. These could include things like environmental impacts, economic impacts, or even population density!

Despite losing one point in the last category, this example synthesis essay is a strong one. It's well-developed, thoughtfully written, and advances an argument on the exam topic using evidence and support throughout.

body-number-four-post-it-note

4 Tips for How to Write a Synthesis Essay

AP Lang is a timed exam, so you have to pick and choose what you want to focus on in the limited time you're given to write the synthesis essay. Keep reading to get our expert advice on what you should focus on during your exam.

Tip 1: Read the Prompt First

It may sound obvious, but when you're pressed for time, it's easy to get flustered. Just remember: when it comes time to write the synthesis essay, read the prompt first !

Why is it so important to read the prompt before you read the sources? Because when you're aware of what kind of question you're trying to answer, you'll be able to read the sources more strategically. The prompt will help give you a sense of what claims, points, facts, or opinions to be looking for as you read the sources.

Reading the sources without having read the prompt first is kind of like trying to drive while wearing a blindfold: you can probably do it, but it's likely not going to end well!

Tip 2: Make Notes While You Read

During the 15-minute reading period at the beginning of the synthesis essay, you'll be reading through the sources as quickly as you can. After all, you're probably anxious to start writing!

While it's definitely important to make good use of your time, it's also important to read closely enough that you understand your sources. Careful reading will allow you to identify parts of the sources that will help you support your thesis statement in your essay, too.

As you read the sources, consider marking helpful passages with a star or check mark in the margins of the exam so you know which parts of the text to quickly re-read as you form your synthesis essay. You might also consider summing up the key points or position of each source in a sentence or a few words when you finish reading each source during the reading period. Doing so will help you know where each source stands on the topic given and help you pick the three (or more!) that will bolster your synthesis argument.

Tip 3: Start With the Thesis Statement

If you don't start your synthesis essay with a strong thesis statement, it's going to be tough to write an effective synthesis essay. As soon as you finish reading and annotating the provided sources, the thing you want to do next is write a strong thesis statement.

According to the CollegeBoard grading guidelines for the AP Lang synthesis essay, a strong thesis statement will respond to the prompt— not restate or rephrase the prompt. A good thesis will take a clear, defensible position on the topic presented in the prompt and the sources.

In other words, to write a solid thesis statement to guide the rest of your synthesis essay, you need to think about your position on the topic at hand and then make a claim about the topic based on your position. This position will either be defending, challenging, or qualifying the claim made in the essay's prompt.

The defensible position that you establish in your thesis statement will guide your argument in the rest of the essay, so it's important to do this first. Once you have a strong thesis statement, you can begin outlining your essay.

Tip 4: Focus on Your Commentary

Writing thoughtful, original commentary that explains your argument and your sources is important. In fact, doing this well will earn you four points (out of a total of six)!

AP Lang provides six to seven sources for you on the exam, and you'll be expected to incorporate quotations, paraphrases, or summaries from at least three of those sources into your synthesis essay and interpret that evidence for the reader.

While incorporating evidence is very important, in order to get the extra point for "sophistication of thought" on the synthesis essay, it's important to spend more time thinking about your commentary on the evidence you choose to incorporate. The commentary is your chance to show original thinking, strong rhetorical skills, and clearly explain how the evidence you've included supports the stance you laid out in your thesis statement.

To earn the 6th possible point on the synthesis essay, make sure your commentary demonstrates a nuanced understanding of the source material, explains this nuanced understanding, and places the evidence incorporated from the sources in conversation with each other. To do this, make sure you're avoiding vague language. Be specific when you can, and always tie your commentary back to your thesis!

body-person-arrows-next

What's Next?

There's a lot more to the AP Language exam than just the synthesis essay. Be sure to check out our expert guide to the entire exam , then learn more about the tricky multiple choice section .

Is the AP Lang exam hard...or is it easy? See how it stacks up to other AP tests on our list of the hardest AP exams .

Did you know there are technically two English AP exams? You can learn more about the second English AP test, the AP Literature exam, in this article . And if you're confused about whether you should take the AP Lang or AP Lit test , we can help you make that decision, too.

Want to improve your SAT score by 160 points or your ACT score by 4 points? We've written a guide for each test about the top 5 strategies you must be using to have a shot at improving your score. Download it for free now:

Get eBook: 5 Tips for 160+ Points

Ashley Sufflé Robinson has a Ph.D. in 19th Century English Literature. As a content writer for PrepScholar, Ashley is passionate about giving college-bound students the in-depth information they need to get into the school of their dreams.

Student and Parent Forum

Our new student and parent forum, at ExpertHub.PrepScholar.com , allow you to interact with your peers and the PrepScholar staff. See how other students and parents are navigating high school, college, and the college admissions process. Ask questions; get answers.

Join the Conversation

Ask a Question Below

Have any questions about this article or other topics? Ask below and we'll reply!

Improve With Our Famous Guides

  • For All Students

The 5 Strategies You Must Be Using to Improve 160+ SAT Points

How to Get a Perfect 1600, by a Perfect Scorer

Series: How to Get 800 on Each SAT Section:

Score 800 on SAT Math

Score 800 on SAT Reading

Score 800 on SAT Writing

Series: How to Get to 600 on Each SAT Section:

Score 600 on SAT Math

Score 600 on SAT Reading

Score 600 on SAT Writing

Free Complete Official SAT Practice Tests

What SAT Target Score Should You Be Aiming For?

15 Strategies to Improve Your SAT Essay

The 5 Strategies You Must Be Using to Improve 4+ ACT Points

How to Get a Perfect 36 ACT, by a Perfect Scorer

Series: How to Get 36 on Each ACT Section:

36 on ACT English

36 on ACT Math

36 on ACT Reading

36 on ACT Science

Series: How to Get to 24 on Each ACT Section:

24 on ACT English

24 on ACT Math

24 on ACT Reading

24 on ACT Science

What ACT target score should you be aiming for?

ACT Vocabulary You Must Know

ACT Writing: 15 Tips to Raise Your Essay Score

How to Get Into Harvard and the Ivy League

How to Get a Perfect 4.0 GPA

How to Write an Amazing College Essay

What Exactly Are Colleges Looking For?

Is the ACT easier than the SAT? A Comprehensive Guide

Should you retake your SAT or ACT?

When should you take the SAT or ACT?

Stay Informed

all ap lang argument essay prompts

Get the latest articles and test prep tips!

Looking for Graduate School Test Prep?

Check out our top-rated graduate blogs here:

GRE Online Prep Blog

GMAT Online Prep Blog

TOEFL Online Prep Blog

Holly R. "I am absolutely overjoyed and cannot thank you enough for helping me!”

How to Write the AP Lang Rhetorical Analysis Essay (With Example)

November 27, 2023

how to write AP Lang rhetorical analysis essay example

Feeling intimidated by the AP Lang Rhetorical Analysis Essay? We’re here to help demystify. Whether you’re cramming for the AP Lang exam right now or planning to take the test down the road, we’ve got crucial rubric information, helpful tips, and an essay example to prepare you for the big day. This post will cover 1) What is the AP Lang Rhetorical Analysis Essay? 2) AP Lang Rhetorical Analysis Rubric 3) AP Lang Rhetorical Analysis: Sample Prompt 4) AP Lang Rhetorical Analysis Essay Example 5)AP Lang Rhetorical Analysis Essay Example: Why It Works

What is the AP Lang Rhetorical Analysis Essay?

The AP Lang Rhetorical Analysis Essay is one of three essays included in the written portion of the AP English Exam. The full AP English Exam is 3 hours and 15 minutes long, with the first 60 minutes dedicated to multiple-choice questions. Once you complete the multiple-choice section, you move on to three equally weighted essays that ask you to synthesize, analyze, and interpret texts and develop well-reasoned arguments. The three essays include:

Synthesis essay: You’ll review various pieces of evidence and then write an essay that synthesizes (aka combines and interprets) the evidence and presents a clear argument. Read our write up on How to Write the AP Lang Synthesis Essay here.

Argumentative essay: You’ll take a stance on a specific topic and argue your case.

Rhetorical essay: You’ll read a provided passage, then analyze the author’s rhetorical choices and develop an argument that explains why the author made those rhetorical choices.

AP Lang Rhetorical Analysis Rubric

The AP Lang Rhetorical Analysis Essay is graded on just 3 rubric categories: Thesis, Evidence and Commentary, and Sophistication . At a glance, the rubric categories may seem vague, but AP exam graders are actually looking for very particular things in each category. We’ll break it down with dos and don’ts for each rubric category:

Thesis (0-1 point)

There’s nothing nebulous when it comes to grading AP Lang Rhetorical Analysis Essay thesis. You either have one or you don’t. Including a thesis gets you one point closer to a high score and leaving it out means you miss out on one crucial point. So, what makes a thesis that counts?

  • Make sure your thesis argues something about the author’s rhetorical choices. Making an argument means taking a risk and offering your own interpretation of the provided text. This is an argument that someone else might disagree with.
  • A good test to see if you have a thesis that makes an argument. In your head, add the phrase “I think that…” to the beginning of your thesis. If what follows doesn’t logically flow after that phrase (aka if what follows isn’t something you and only you think), it’s likely you’re not making an argument.
  • Avoid a thesis that merely restates the prompt.
  • Avoid a thesis that summarizes the text but does not make an argument.

Evidence and Commentary (0-4 points)

This rubric category is graded on a scale of 0-4 where 4 is the highest grade. Per the AP Lang Rhetorical Analysis rubric, to get a 4, you’ll want to:

  • Include lots of specific evidence from the text. There is no set golden number of quotes to include, but you’ll want to make sure you’re incorporating more than a couple pieces of evidence that support your argument about the author’s rhetorical choices.
  • Make sure you include more than one type of evidence, too. Let’s say you’re working on your essay and have gathered examples of alliteration to include as supporting evidence. That’s just one type of rhetorical choice, and it’s hard to make a credible argument if you’re only looking at one type of evidence. To fix that issue, reread the text again looking for patterns in word choice and syntax, meaningful figurative language and imagery, literary devices, and other rhetorical choices, looking for additional types of evidence to support your argument.
  • After you include evidence, offer your own interpretation and explain how this evidence proves the point you make in your thesis.
  • Don’t summarize or speak generally about the author and the text. Everything you write must be backed up with evidence.
  • Don’t let quotes speak for themselves. After every piece of evidence you include, make sure to explain your interpretation. Also, connect the evidence to your overarching argument.

Sophistication (0-1 point)

In this case, sophistication isn’t about how many fancy vocabulary words or how many semicolons you use. According to College Board , one point can be awarded to AP Lang Rhetorical Analysis essays that “demonstrate sophistication of thought and/or a complex understanding of the rhetorical situation” in any of these three ways:

  • Explaining the significance or relevance of the writer’s rhetorical choices.
  • Explaining the purpose or function of the passage’s complexities or tensions.
  • Employing a style that is consistently vivid and persuasive.

Note that you don’t have to achieve all three to earn your sophistication point. A good way to think of this rubric category is to consider it a bonus point that you can earn for going above and beyond in depth of analysis or by writing an especially persuasive, clear, and well-structured essay. In order to earn this point, you’ll need to first do a good job with your thesis, evidence, and commentary.

  • Focus on nailing an argumentative thesis and multiple types of evidence. Getting these fundamentals of your essay right will set you up for achieving depth of analysis.
  • Explain how each piece of evidence connects to your thesis.
  • Spend a minute outlining your essay before you begin to ensure your essay flows in a clear and cohesive way.
  • Steer clear of generalizations about the author or text.
  • Don’t include arguments you can’t prove with evidence from the text.
  • Avoid complex sentences and fancy vocabulary words unless you use them often. Long, clunky sentences with imprecisely used words are hard to follow.

AP Lang Rhetorical Analysis: Sample Prompt

The sample prompt below is published online by College Board and is a real example from the 2021 AP Exam. The prompt provides background context, essay instructions, and the text you need to analyze. For sake of space, we’ve included the text as an image you can click to read. After the prompt, we provide a sample high scoring essay and then explain why this AP Lang Rhetorical Analysis essay example works.

Suggested time—40 minutes.

(This question counts as one-third of the total essay section score.)

On February 27, 2013, while in office, former president Barack Obama delivered the following address dedicating the Rosa Parks statue in the National Statuary Hall of the United States Capitol building. Rosa Parks was an African American civil rights activist who was arrested in 1955 for refusing to give up her seat on a segregated bus in Montgomery, Alabama. Read the passage carefully. Write an essay that analyzes the rhetorical choices Obama makes to convey his message.

In your response you should do the following:

  • Respond to the prompt with a thesis that analyzes the writer’s rhetorical choices.
  • Select and use evidence to support your line of reasoning.
  • Explain how the evidence supports your line of reasoning.
  • Demonstrate an understanding of the rhetorical situation.
  • Use appropriate grammar and punctuation in communicating your argument.

AP Lang Rhetorical Analysis Essay Example

In his speech delivered in 2013 at the dedication of Rosa Park’s statue, President Barack Obama acknowledges everything that Parks’ activism made possible in the United States. Telling the story of Parks’ life and achievements, Obama highlights the fact that Parks was a regular person whose actions accomplished enormous change during the civil rights era. Through the use of diction that portrays Parks as quiet and demure, long lists that emphasize the extent of her impacts, and Biblical references, Obama suggests that all of us are capable of achieving greater good, just as Parks did.

Although it might be a surprising way to start to his dedication, Obama begins his speech by telling us who Parks was not: “Rosa Parks held no elected office. She possessed no fortune” he explains in lines 1-2. Later, when he tells the story of the bus driver who threatened to have Parks arrested when she refused to get off the bus, he explains that Parks “simply replied, ‘You may do that’” (lines 22-23). Right away, he establishes that Parks was a regular person who did not hold a seat of power. Her protest on the bus was not part of a larger plan, it was a simple response. By emphasizing that Parks was not powerful, wealthy, or loud spoken, he implies that Parks’ style of activism is an everyday practice that all of us can aspire to.

AP Lang Rhetorical Analysis Essay Example (Continued)

Even though Obama portrays Parks as a demure person whose protest came “simply” and naturally, he shows the importance of her activism through long lists of ripple effects. When Parks challenged her arrest, Obama explains, Martin Luther King, Jr. stood with her and “so did thousands of Montgomery, Alabama commuters” (lines 27-28). They began a boycott that included “teachers and laborers, clergy and domestics, through rain and cold and sweltering heat, day after day, week after week, month after month, walking miles if they had to…” (lines 28-31). In this section of the speech, Obama’s sentences grow longer and he uses lists to show that Parks’ small action impacted and inspired many others to fight for change. Further, listing out how many days, weeks, and months the boycott lasted shows how Parks’ single act of protest sparked a much longer push for change.

To further illustrate Parks’ impact, Obama incorporates Biblical references that emphasize the importance of “that single moment on the bus” (lines 57-58). In lines 33-35, Obama explains that Parks and the other protestors are “driven by a solemn determination to affirm their God-given dignity” and he also compares their victory to the fall the “ancient walls of Jericho” (line 43). By of including these Biblical references, Obama suggests that Parks’ action on the bus did more than correct personal or political wrongs; it also corrected moral and spiritual wrongs. Although Parks had no political power or fortune, she was able to restore a moral balance in our world.

Toward the end of the speech, Obama states that change happens “not mainly through the exploits of the famous and the powerful, but through the countless acts of often anonymous courage and kindness” (lines 78-81). Through carefully chosen diction that portrays her as a quiet, regular person and through lists and Biblical references that highlight the huge impacts of her action, Obama illustrates exactly this point. He wants us to see that, just like Parks, the small and meek can change the world for the better.

AP Lang Rhetorical Analysis Essay Example: Why It Works

We would give the AP Lang Rhetorical Analysis essay above a score of 6 out of 6 because it fully satisfies the essay’s 3 rubric categories: Thesis, Evidence and Commentary, and Sophistication . Let’s break down what this student did:

The thesis of this essay appears in the last line of the first paragraph:

“ Through the use of diction that portrays Parks as quiet and demure, long lists that emphasize the extent of her impacts, and Biblical references, Obama suggests that all of us are capable of achieving greater good, just as Parks did .”

This student’s thesis works because they make a clear argument about Obama’s rhetorical choices. They 1) list the rhetorical choices that will be analyzed in the rest of the essay (the italicized text above) and 2) include an argument someone else might disagree with (the bolded text above).

Evidence and Commentary:

This student includes substantial evidence and commentary. Things they do right, per the AP Lang Rhetorical Analysis rubric:

  • They include lots of specific evidence from the text in the form of quotes.
  • They incorporate 3 different types of evidence (diction, long lists, Biblical references).
  • After including evidence, they offer an interpretation of what the evidence means and explain how the evidence contributes to their overarching argument (aka their thesis).

Sophistication

This essay achieves sophistication according to the AP Lang Rhetorical Analysis essay rubric in a few key ways:

  • This student provides an introduction that flows naturally into the topic their essay will discuss. Before they get to their thesis, they tell us that Obama portrays Parks as a “regular person” setting up their main argument: Obama wants all regular people to aspire to do good in the world just as Rosa Parks did.
  • They organize evidence and commentary in a clear and cohesive way. Each body paragraph focuses on just one type of evidence.
  • They explain how their evidence is significant. In the final sentence of each body paragraph, they draw a connection back to the overarching argument presented in the thesis.
  • All their evidence supports the argument presented in their thesis. There is no extraneous evidence or misleading detail.
  • They consider nuances in the text. Rather than taking the text at face value, they consider what Obama’s rhetorical choices imply and offer their own unique interpretation of those implications.
  • In their final paragraph, they come full circle, reiterate their thesis, and explain what Obama’s rhetorical choices communicate to readers.
  • Their sentences are clear and easy to read. There are no grammar errors or misused words.

AP Lang Rhetorical Analysis Essay—More Resources

Looking for more tips to help your master your AP Lang Rhetorical Analysis Essay? Brush up on 20 Rhetorical Devices High School Students Should Know and read our Tips for Improving Reading Comprehension . If you’re ready to start studying for another part of the AP English Exam, find more expert tips in our How to Write the AP Lang Synthesis blog post.

Considering what other AP classes to take? Read up on the Hardest AP Classes .

  • High School Success

' src=

Christina Wood

Christina Wood holds a BA in Literature & Writing from UC San Diego, an MFA in Creative Writing from Washington University in St. Louis, and is currently a Doctoral Candidate in English at the University of Georgia, where she teaches creative writing and first-year composition courses. Christina has published fiction and nonfiction in numerous publications, including The Paris Review , McSweeney’s , Granta , Virginia Quarterly Review , The Sewanee Review , Mississippi Review , and Puerto del Sol , among others. Her story “The Astronaut” won the 2018 Shirley Jackson Award for short fiction and received a “Distinguished Stories” mention in the 2019 Best American Short Stories anthology.

  • 2-Year Colleges
  • Application Strategies
  • Best Colleges by Major
  • Big Picture
  • Career & Personality Assessment
  • College Essay
  • College Search/Knowledge
  • College Success
  • Costs & Financial Aid
  • Dental School Admissions
  • Extracurricular Activities
  • Graduate School Admissions
  • High Schools
  • Law School Admissions
  • Medical School Admissions
  • Navigating the Admissions Process
  • Online Learning
  • Private High School Spotlight
  • Summer Program Spotlight
  • Summer Programs
  • Test Prep Provider Spotlight

College Transitions Sidebar Block Image

“Innovative and invaluable…use this book as your college lifeline.”

— Lynn O'Shaughnessy

Nationally Recognized College Expert

College Planning in Your Inbox

Join our information-packed monthly newsletter.

Sign Up Now

IMAGES

  1. AP Lang Argumentative Essay Example- Prompt: Thomas Paine's America

    all ap lang argument essay prompts

  2. AP Lang Argument Essay Example Unknown

    all ap lang argument essay prompts

  3. Incredible Ap Lang Argument Essay Prompts ~ Thatsnotus

    all ap lang argument essay prompts

  4. How to Ace the AP Language Argument Essay

    all ap lang argument essay prompts

  5. 010 Essay Example Ap Lang Argument Prompts Types Of Exam Study ~ Thatsnotus

    all ap lang argument essay prompts

  6. 6 AP Language and Composition Argument & "Suprise" Prompts Plus Tips

    all ap lang argument essay prompts

VIDEO

  1. How to NAIL the UC Essay Prompts (pt. 3)

  2. How to NAIL the UCLA Essay Prompts (pt. 1)

  3. Ap Lang video essay: censorship

  4. AP Lang

  5. AmStud

  6. AP Lang Rhetorical Analysis Rubric

COMMENTS

  1. AP English Language and Composition Past Exam Questions

    Download free-response questions from past exams along with scoring guidelines, sample responses from exam takers, and scoring distributions. If you are using assistive technology and need help accessing these PDFs in another format, contact Services for Students with Disabilities at 212-713-8333 or by email at [email protected]. Note ...

  2. How to Write the AP Lang Argument Essay + Examples

    2. Pick one side of the argument, but acknowledge the other side. When you write the essay, it's best if you pick one side of the debate and stick with it for the entire essay. All your evidence should be in support of that one side. However, in your introductory paragraph, as you introduce the debate, be sure to mention any merit the ...

  3. How to Write the AP Lang Argument Essay (With Example)

    Her story "The Astronaut" won the 2018 Shirley Jackson Award for short fiction and received a "Distinguished Stories" mention in the 2019 Best American Short Stories anthology. Ap Lang Argumentative Essay - Expert advice on how to pen a winning essay + an AP Lang argument essay example to guide your writing.

  4. PDF AP Language Argument Prompts (some adaptions)

    Asher AP ELAC Past AP Language ARGUMENT Prompts (some adaptations) 2012 Consider the distinct perspective expressed in the following statements: "If you develop the absolute sense of certainty that powerful beliefs provide, then you can get yourself to accomplish virtually anything, including those things that other people are certain are impossible." (William Phelps )

  5. Crafting an Impressive Argumentative Essay for AP Lang

    4. Organize Your Essay Effectively: - Tip: Structure your essay with a clear introduction, body paragraphs, and a conclusion. Ensure a logical flow of ideas, with each paragraph contributing to the overall argument. 5. Provide Context and Background: - Tip: Begin with a brief introduction that provides context for your argument.

  6. PDF AP English Language and Composition Question 3: Argument (2019) Sample

    AP English Language and Composition Question 3: Argument (2019) Sample Student Responses 3 harmful. It is possible to have free enterprise without sacrificing freedom, even if it is rarely seen nowadays. One should be able to criticize capitalism without entirely, writing it off. After all, if a system cannot be challenged, how can it improve?

  7. PDF AP English Language and Composition

    In your response you should do the following: Respond to the prompt with a thesis that presents a defensible position. Provide evidence to support your line of reasoning. Explain how the evidence supports your line of reasoning. Use appropriate grammar and punctuation in communicating your argument. 2023 College Board.

  8. Understanding the AP® English Language Argument Rubric

    Utilizing Logic and Details. Supporting details and logical arguments are a key point in the AP® English Language argument essay rubric, because lending more support to your argument allows the examiners to buy into that argument. When the examiners see your point so nicely developed, then you will jump up to higher scores such as 7s, 8s, or ...

  9. How to Craft an Argument for AP® English Language

    How the AP® English Language Persuasive Essay Works. Persuasion through essay writing is something you probably learned about a long time ago, but the AP® English Language Exam's persuasion essay requires some more specific tips. You will be given a prompt that may or may not reference a reading sample; it will ask you to then "defend ...

  10. How to Get a 6 on Argument FRQ in AP® English Language

    Pick an opinion and stick to it. Choose one side of the argument and one clear claim to support all the way through. Craft a thesis statement. Your thesis should be clear, concise, and introduce the content of your essay. Craft a chronological argument. Make an argument that builds on its prior points.

  11. PDF AP English Language and Composition 2018 Free-Response Questions

    2018 AP. ® ENGLISH LANGUAGE AND COMPOSITION FREE-RESPONSE QUESTIONS. ENGLISH LANGUAGE AND COMPOSITION. SECTION II. Total time —2 hours and 15 minutes. Question 1. Suggested reading and writing time —55 minutes. It is suggested that you spend 15 minutes reading the question, analyzing and evaluating the sources, and 40 minutes writing your ...

  12. Expert Guide to the AP Language and Composition Exam

    One essay where you create an original argument in response to a prompt. ... Unlike on some other exams, where the content is the most important aspect of the essay, on the AP Language Exam, organization, a well-developed argument, and strong evidence are all critical to strong essay scores.

  13. Every AP English Language and Composition Practice Exam

    The AP Language and Composition exam has two sections: a multiple-choice section with 45 questions, and a free-response section with three essay questions—one synthesis prompt, one analysis prompt, and one argument prompt. But not all AP Lang practice tests are like the real exam, and they aren't all of equal quality.

  14. PDF MsEffie's List of Advanced Placement® Language and Composition Prompts

    Evaluate his argument. (Excerpt included) Analyze the rhetorical techniques Ellen Goodman uses to convey her attitude toward Phil, the subject of her piece, "The Company Man." After reading his paragraph, defend, challenge, or qualify James Baldwin's ideas about the importance of language as a "key to identity" and social acceptance.

  15. PDF AP English Language and Composition

    AP® English Language and Composition 2022 Scoring Guidelines. Argument Essay 6 points . Colin Powell, a four-star general and former United States secretary of state, wrote in his 1995 autobiography: "[W]e do not have the luxury of collecting information indefinitely. At some point, before we can have every possible fact in hand, we have to ...

  16. PDF AP English Language and Composition Writing the Persuasive Essay

    Writing the Persuasive Essay: Assembling an Argument Student Activity Introduction One third of the AP* English Language and Composition Exam will require you to write a persuasive essay. The good news is that this is your opportunity to use all of the skills and clever tactics that you have learned from reading established writers all year long.

  17. What's the best approach for the AP Lang argument essay?

    The AP Lang argument essay is all about making a strong, well-structured argument in a clear and concise manner. Here's a step-by-step approach you can follow and some tips to help you prepare: 1. Understand the prompt: Carefully read the given prompt and make sure you understand what it's asking you to do. Keep an eye out for key terms that ...

  18. AP Lang Exam Guide

    1 argument essay. Scoring Rubric for the 2024 AP Lang Essays. Synthesis Essay. Thesis. 1 point for a defensible thesis that responds to the prompt. Evidence and Commentary. Max of 4 points for providing evidence from at least 3 sources that support the line of reasoning AND commentary that explains and analyzes the evidence. Sophistication

  19. SAT® and AP® English Language essay prompts

    Click below to view a list of official SAT® Essay prompts. Then, click the title of each prompt to view the page or download a PDF of the text. 💡Some prompts below include links to an external resource. To include this link as your essay prompt when creating a writing assignment in NoRedInk, be sure to copy and paste the website's full ...

  20. How to Write a Perfect Synthesis Essay for the AP Language Exam

    Paragraph 1: The prompt presents and briefly explains the topic that you'll be writing your synthesis essay about. That topic is the concept of eminent domain. Paragraph 2: The prompt presents a specific claim about the concept of eminent domain in this paragraph: Eminent domain is productive and beneficial.This paragraph instructs you to decide whether you want to defend, challenge, or ...

  21. How to Write the AP Lang Rhetorical Analysis Essay (With Example)

    Her story "The Astronaut" won the 2018 Shirley Jackson Award for short fiction and received a "Distinguished Stories" mention in the 2019 Best American Short Stories anthology. How to write the AP Lang rhetorical analysis essay. We look at a AP lang rhetorical analysis essay example and explore do's and don'ts.